In need help with the answer

In Need Help With The Answer

Answers

Answer 1

Answer:

2

Step-by-step explanation:

there is 2:   3  1  

                      4


Related Questions

What is the volume of a sphere with a diameter of 57.1 cm, rounded to the nearest tenth of a cubic centimeter?

Answers

Answer:

so we have to use the formula 4/3PiR^3

which if we do we get the volume of

V≈97478.08

Answer:

V≈97478.08cm³

Step-by-step explanation:

Using the formulas

V=4/3πr3

d=2r

V=1/6πd3

D=1

1/6·πd3 1/6π ·57.13≈97478.07565cm³

I hope this helps. I worked hard on this one.

Triple the sum of two and a number.

Answers

Answer: 3(2+x) or 3x+6

Step-by-step explanation:

Take this a step at a time

First is a sum of two and a number

set "a number" as x

sum of two and a number would be 2+x

triple it

3(2+x) or 3x+6

Answer:

3(x+2)

Step-by-step explanation:

Explanation:

The " sum of a number and 2

" means ADD them:

→ x+2

Triple that means multiply by

3

→ 3(x+2)

10t+[tex]\geq[/tex]130+3.5t

Answers

Answer:

The answer is [tex]t\geq 20[/tex].

Step-by-step explanation:

To solve the inequality, start by solving for the variable [tex]t[/tex].

To solve for the variable [tex]t[/tex], subtract [tex]3.5t[/tex] from both sides. The inequality will look like [tex]6.5t\geq 130[/tex].

Then, divide both sides by 6.5 in order to get the variable [tex]t[/tex] by itself. The inequality answer will look like [tex]t\geq 20[/tex].

Consider the following sets of sample data: A: 20,347, 20,327, 22,117, 21,762, 20,864, 20,102, 21,684, 20,063, 21,728, 21,580, 21,720, 20,920, 21,442, 20,766 B: 3.38, 4.64, 4.09, 3.93, 4.25, 4.63, 4.78, 4.25 Which of the above sets of sample data has the larger spread

Answers

Answer:

Data B

Step-by-step explanation:

Given the data :

A: 20347, 20327, 22117, 21762, 20864, 20102, 21684, 20063, 21728, 21580, 21720, 20920, 21442, 20766

B: 3.38, 4.64, 4.09, 3.93, 4.25, 4.63, 4.78, 4.25

The spread of a data gives the variation in the data values of a given sample.

To obtain which data has the larger spread, we obtain the coefficient of variation. Which is the ratio of the standard deviation and the mean of the dataset.

(Standard deviation / mean) * 100%

Using calculator :

Data A :

Mean, x = 21101.5714

Standard deviation, s = 700.28925

Coefficient of Variation :

(700.28925 / 21101.5714) * 100% = 3.32%

Data B :

Mean, x = 4.24375

Standard deviation, s = 0.457006955

Coefficient of Variation :

(0.457006955 / 4.24375) * 100% = 10.77%

10.77% > 3.32%

Hence. Data B has a larger spread

please help zkhdusjdushs​

Answers

Answer:

48

Step-by-step explanation:

16×3=48

therefore the answer is forty eight

NEED HELP ASAP
Which equation does the graph of the systems of equations solve?

two linear functions intersecting at 3, negative 2

−one thirdx + 3 = x − 1
one thirdx − 3 = −x + 1
−one thirdx + 3 = −x − 1
one thirdx + 3 = x − 1

Answers

Answer:

[tex]\frac{1}{3}x-3=-x+1[/tex]

Step-by-step explanation:

Since there are plenty of lines that can pass through a single point, he only way to solve this question is by substituting values of [tex]3[/tex] into each equation and seeing if both equations return a value of [tex]-2[/tex].

Starting with the first answer choice:

[tex]-\frac{1}{3}(3)+3=3-1,\\-1+3=3-1,\\2=2[/tex]

Since none of the equations here return a value of [tex]-2[/tex], the correct answer must not include [tex]-\frac{1}{3}x+3[/tex] or [tex]x-1[/tex].

Thus, we can eliminate answer choices A, C, and D, hence the correct answer is [tex]\boxed{\text{B. }\frac{1}{3}x-3=-x+1}[/tex]

PLZ Help. I have tried so many ways. I will give Brainiest! 30 points for the correct answer! Help!

Answers

Answer:

1=(w=6,C=20.80

2=(W=2,C=27.05)

3=(w=11,C=14.25)

4=(w=15,C=11.05)

Step-by-step explanation:

Just do the work

0.55x15=8.25+2.80=11.05

1.10x6=6.6+14.20=20.8

11x0.65=7.15+7.10=14.25

Out of process of elimination, 2=(W=2,C=27.05)

Answer:

Below.

Step-by-step explanation:

The second option on the bottom matches the first of the top  list.

This is how we check:

w = 6 and C = 20 .80

Plug these into the top equation:

C = 14.2 + 1.10w

20.80 = 14.2 + 6(1.10) = 14.20 + 6.60 = 20.80.

So that proves that we have the right answer.

The last one on the bottom matches the second on the top list.

The third option ion the bottom matches the third equation on the top

.

The first one on bottom matches the last one the top.

The last 3 are answered in exactly the same way as the first.

A car rental company charges an initial fee plus a constant fee per kilometer driven.

The table compares the total distance driven on the trip (in kilometers) and the price of the rental (in dollars).

Distance (kilometers) Price (dollars)

105 183

140 204

175 225

What is the company's initial fee?

Answers

Answer:

$120

Step-by-step explanation:

constant fee = (204-183) /(140-105)

= 21/35 = $0.6 per km

so, initial fee =

183 - (105 × 0.6)

= 183 - 63

= 120

Answer:

120

Step-by-step explanation:

trust me.

Which figures always have right angles?

parallelogram
square
rectangle
trapezoid

Answers

9514 1404 393

Answer:

squarerectangle

Step-by-step explanation:

A rectangle is a parallelogram that has right angles. A square is a rectangle that has adjacent sides the same length.

The figures that always have right angles are ...

  square

  rectangle

y
.
4
3
2
1-
2
-4
-3
-2
-1
1
2
3
4
-1
-2
-3
-4
What is the slope of the line?

Answers

Answer:

-1

Step-by-step explanation:

Slope is rise over run

graph us going up by 2 units and going to left by 2 units

[tex]\frac{2}{ - 2} \\ - 1[/tex]

i need help with this geometry question pleasee

Answers

Answer:i dont know

Step-by-step explanation:lol

Answer:

tmabm queria saber

Step-by-step explanation:

the students in charge of the class booth at a carnival would like to earn $3 for every item they sell. they spent $55 for the materials to make the items. solve the inequality 3x-55_>65 which represents how many items they need to sell to make profit of at least $65

Answers

Answer:

x ≥ 40

Step-by-step explanation:

3x - 55 ≥ 65

combine like terms

3x  ≥ 65 + 55

3x  ≥120

divide both sides of the equation by 3

x  ≥ 40

Cary calculated the surface area of a box in the shape of a rectangular prism. She wrote the equation 148 = 2 (6w + 6h + hw) to represent the width and height of the box. She solved for w and got w = StartFraction 74 minus 6 h Over h + 6 EndFraction Which of the following is an equivalent equation?

Answers

Answer:

the answer is w = 148-12h/12+2h

The equivalent equation is -

[tex]$w=\frac{74-6h}{h+6}[/tex]

We have the equation written by Carly → 148 = 2 (6w + 6h + hw) that represent the width and height of the box in the shape of a rectangular prism.

We have to solve for w.

What do you mean by Equivalent expression ?

Any expression written in a form different from the original form, but gives same result for any input are called equivalent expressions.

Solve for x :  [tex]$log(\frac{x}{\omega}) = 2\pi[/tex]

We have -  

[tex]$log(\frac{x}{\omega}) = 2\pi[/tex]

log(x) - log(ω) = 2π

log(x) = 2π + log(ω)

x = [tex]e^{(2\pi + log(\omega))} = e^{2\pi } \times e^{log(\omega)}[/tex]

According to the question, we have -

148 = 2 (6w + 6h + hw)

(6w + 6h + hw) = 74

hw + 6w = 74  - 6h

w(h + 6) = 74 - 6h

[tex]$w=\frac{74-6h}{h+6}[/tex]

Hence, the equivalent equation is -

[tex]$w=\frac{74-6h}{h+6}[/tex]

To solve more questions on Rearranging expression, visit the link below-

https://brainly.com/question/1824488

#SPJ2

Find the area of the triangle.
18 in.
25 in.

Answers

Answer: i hope you pass that quiz

Step-by-step explanation:

25 x 18 = 450
450 / 2 = 225
The answer is 225in squared
The formula for a right angled triangle is
area = base x height / 2

ABC is a right angled triangle. if B = 90°, AC = 96 cm, C = 30°.

AB =
cm

Answers

Answer:

48 ANS.

MAY IT HELPED U

PLEASE HELP ASAP
Write the equation that represents the trigonometric graph

Answers

Answer(s):

[tex]\displaystyle y = 300sin\:(\frac{\pi}{8}x + \frac{\pi}{2}) + 400 \\ y = 300cos\:\frac{\pi}{8}x + 400[/tex]

Step-by-step explanation:

[tex]\displaystyle y = Asin(Bx - C) + D \\ \\ Vertical\:Shift \hookrightarrow D \\ Horisontal\:[Phase]\:Shift \hookrightarrow \frac{C}{B} \\ Wavelength\:[Period] \hookrightarrow \frac{2}{B}\pi \\ Amplitude \hookrightarrow |A| \\ \\ Vertical\:Shift \hookrightarrow 400 \\ Horisontal\:[Phase]\:Shift \hookrightarrow \frac{C}{B} \hookrightarrow \boxed{-4} \hookrightarrow \frac{-\frac{\pi}{2}}{\frac{\pi}{8}} \\ Wavelength\:[Period] \hookrightarrow \frac{2}{B}\pi \hookrightarrow \boxed{16} \hookrightarrow \frac{2}{\frac{\pi}{8}}\pi \\ Amplitude \hookrightarrow 300[/tex]

OR

[tex]\displaystyle y = Acos(Bx - C) + D \\ \\ Vertical\:Shift \hookrightarrow D \\ Horisontal\:[Phase]\:Shift \hookrightarrow \frac{C}{B} \\ Wavelength\:[Period] \hookrightarrow \frac{2}{B}\pi \\ Amplitude \hookrightarrow |A| \\ \\ Vertical\:Shift \hookrightarrow 400 \\ Horisontal\:[Phase]\:Shift \hookrightarrow 0 \\ Wavelength\:[Period] \hookrightarrow \frac{2}{B}\pi \hookrightarrow \boxed{16} \hookrightarrow \frac{2}{\frac{\pi}{8}}\pi \\ Amplitude \hookrightarrow 300[/tex]

You will need the above information to help you interpret the graph. First off, keep in mind that although this looks EXACTLY like the cosine graph, if you plan on writing your equation as a function of sine, then there WILL be a horisontal shift, meaning that a C-term will be involved. As you can see, the photograph on the right displays the trigonometric graph of [tex]\displaystyle y = 300sin\:\frac{\pi}{8}x + 400,[/tex]in which you need to replase "cosine" with "sine", then figure out the appropriate C-term that will make the graph horisontally shift and map onto the sine graph [photograph on the left], accourding to the horisontal shift formula above. Also keep in mind that the −C gives you the OPPOCITE TERMS OF WHAT THEY REALLY ARE, so you must be careful with your calculations. So, between the two photographs, we can tell that the sine graph [photograph on the right] is shifted [tex]\displaystyle 4\:units[/tex]to the right, which means that in order to match the cosine graph [photograph on the left], we need to shift the graph BACKWARD [tex]\displaystyle 4\:units,[/tex]which means the C-term will be negative, and by perfourming your calculations, you will arrive at [tex]\displaystyle \boxed{-4} = \frac{-\frac{\pi}{2}}{\frac{\pi}{8}}.[/tex]So, the sine graph of the cosine graph, accourding to the horisontal shift, is [tex]\displaystyle y = 300sin\:(\frac{\pi}{8}x + \frac{\pi}{2}) + 400.[/tex]Now, with all that being said, in this case, sinse you ONLY have a graph to wourk with, you MUST figure the period out by using wavelengths. So, looking at where the graph WILL hit [tex]\displaystyle [36, 400],[/tex]from there to [tex]\displaystyle [20, 400],[/tex]they are obviously [tex]\displaystyle 16\:units[/tex]apart, telling you that the period of the graph is [tex]\displaystyle 16.[/tex]Now, the amplitude is obvious to figure out because it is the A-term, but of cource, if you want to be certain it is the amplitude, look at the graph to see how low and high each crest extends beyond the midline. The midline is the centre of your graph, also known as the vertical shift, which in this case the centre is at [tex]\displaystyle y = 400,[/tex]in which each crest is extended three hundred units beyond the midline, hence, your amplitude. So, no matter how far the graph shifts vertically, the midline will ALWAYS follow.

I am delighted to assist you at any time.

Add - 3/x + 7y/x .
-4y/2x
-3 + 7y/x
- 10y/x
-3 + 7y/2x

Answers

Answer:

[tex]-\frac{3}{x} + \frac{7y}{x} = \frac{-3+ 7y}{x}[/tex]

Step-by-step explanation:

Given

[tex]-\frac{3}{x} , \frac{7y}{x}[/tex]

Required

Add

The statement can be interpreted as:

[tex]-\frac{3}{x} + \frac{7y}{x}[/tex]

Take LCM

[tex]-\frac{3}{x} + \frac{7y}{x} = \frac{-3+ 7y}{x}[/tex]

I WILL MARK BRAINLIEST!


Two identical rubber balls are dropped from different heights. Ball 1 is dropped from
a height of 110 feet, and ball 2 is dropped from a height of 276 feet. Use the
function f(t) -16t2 + h to determine the current height, f(t), of a ball dropped from a
height h, over given time t.
Write a function for the height of ball 1.
hi(t)

Answers

9514 1404 393

Answer:

  h₁(t) = -16t² +110

Step-by-step explanation:

Put the given initial height into the given formula. That will give the requested function. If the function name is supposed to be h₁(t), then rename it.

  f(t) = -16t² +110 . . . . . . h = 110, the initial height

  h₁(t) = -16t² +110

determine the missing angle.

Answers

Answer:

the missing angle will be 140°

Answer: 140

Step-by-step explanation:


Francis borrowed $20 from his dad in the morning. Later, he gave his dad 8
dollars back. What rational number represents the overall amount of money Francis still owes his dad?

Answers

Answer:

13

Step-by-step explanation:

Francis borrowed 20$ and gives his dad 8$ he owes 12$ I hope it helps :)

The answer would be $12 still owed to his father.

someone help me on this plzzz!!

Answers

Answer:

y = (3/2)x + 2

Step-by-step explanation:

The equation of a straight line is given by:

y = mx + b;

where y, x are variables, m is the slope of the line and b is the y intercept (value of y when x is 0).

From the graph, we can see that the y intercept is 2 units. Also, the line passes through point (2, 5) and (-2, -1). Therefore the slope of the line is:

[tex]slope(m)=\frac{y_2-y_1}{x_2-x_1}=\frac{-1-5}{-2-2}=\frac{3}{2}[/tex]

Using the equation a straight line:

y = mx + b; substituting gives:

y = (3/2)x + 2

A large soda bottle cap has a radius of 1+2 /2 centimeters. What is the area of the bottle cap? (R the area of a circle is no where r is the radius). 1 + 2v2 A. (5-12 - square IT square centimeters​

Answers

Answer:

bottle top

Step-by-step explanation:

Find the solution to the equations.
2х-у=-3
х+у=0

0,0

-1,1

1,-1

Answers

It’s 0,0. The correct answer!

Answer:

Option B : ( - 1 , 1 )   ==>    x = -1 , y = 1

Step-by-step explanation:

2x - y = - 3              -------- ( 1 )

x + y = 0 => x = - y   -------( 2 )

Substitute ( 2 ) in ( 1 ) : 2x  - y = - 3

                                 2( - y ) - y = - 3

                                  - 2y - y = -  3

                                   - 3y = - 3

                                       y = 1

Substitute y in ( 2) : x + y = 0

                              x + 1 = 0

                                x =  - 1

Plz help I’m suck on this question!!!!!!

Answers

Answer:

U = 1

V = -5

Step-by-step explanation:

U = -1 - (-2) = 1

V = -4 -1 = -5

U: 1
V: -5

Explanation: Take the a & b values of that row and subtract them

The sides of two cubes are in the ratio 2:5, what is the ratio of their volumes?

Answers

9514 1404 393

Answer:

  8 : 125

Step-by-step explanation:

The volume is proportional to the cube of the side lengths. Hence, the ratio of volumes is proportional to the cube of the ratio of side lengths.

  v1 : v2 = (2 : 5)^3 = 2^3 : 5^3 = 8 : 125

120+12314543-900-90+12

Answers

Answer:

12313685 is the answer............happy to help u

12,313,685 !!! happy
to help

The center of a circle is at (6,-7) and the diameter of the circle is 22. Which of the following is the equation of the circle? (Part 1 and Part 2)

Answers

Answer:

P1.H.x²+y²=34

centre[h,k]=(0,0)

point=[3,5]

now

radius=[tex]\sqrt{(0-3)²+(0-5)²}=\sqrt{34}[/tex]

now

equation of a circle is;

(x-h)²+(y-k)²=[tex]\sqrt{34}²[/tex]

x²+y²=34

P:2I.(x-6)²+(y+7)²=121

centre[h,k]=(6,-7)

diameter=22

radius[r]=22/2=11

now

equation of a circle is;

(x-h)²+(y-k)²=r²

(x-6)²+(y+7)²=11²

(x-6)²+(y+7)²=121

PLEASE HELP!!!

WILL MARK BRAINLIEST!!!!


In the triangle shown, solve for X to the nearest degree.

Multiple choice!

Thank you!

Answers

Answer:

x=42°

Step-by-step explanation:

take x degree as reference angle

using tan rule

tan x=opposite/adjacent

tan x=9/10

tan x=0.9

x=[tex]tan^{-1}[/tex]0.9

x=41.98

x=42 degree

Answer:

41.987° which rounds to 42 degrees

Step-by-step explanation:

What is the value of (–7 + 3i) – (2 – 6i)?
–9 + 9i
–9 – 3i
–5 – 3i
–5 + 9i

Answers

Answer:

- 9 + 9i

Step-by-step explanation:

(- 7 + 3i) - (2 - 6i)

- 7 + 3i - 2 + 6i

- 7 - 2 + 9i

- 9 + 9i

The value of(-7+3i)-(2-6i)
-9+9i

Sara bought a 24-ounce can of tomato sauce for $8.40. What is the unit price per ounce?

Answers

Answer:

$0.35

Step-by-step explanation:

8.40/24 = $0.35

Other Questions
A transition metal in the fourth period from the following list : Cu, O , Pr, Ag What value of x is in the solution set of 2(3x - 1) > 4x - 6? 2 He.... my tablet for months before he decided to buy his own. a had been borrowing b had borrowed C borrowed d borrows The table represents the speed of a car in a northern direction over several seconds. Column 1 would be on the x-axis, and Column 2 would be on the y-axis. Which best lists the titles of each column? Column 1 0 2 4. 6 8 10 Column 2 5 10 15 20 25 30 Column 1 should be titled "Time," and Column 2 should be titled "Velocity." Column 1 should be titled "Velocity," and Column 2 should be titled "Time." Column 1 should be titled "Time," and Column 2 should be titled "Acceleration." O Column 1 should be titled "Acceleration," and Column 2 should be titled "Time." Writers editing sentences for errors in grammar and usage would look forA. Problems with verb tenses.B. Misused or missing capitalization.C. Sentences missing commas or periods.D. Ways to replace overused words. Analysis of Adjusting Entry for Supplies: Analyze each situation and indicate the correct dollar amount for the adjusting entry. (Trial balance is abbreviated as TB.)1. Ending inventory of supplies is $239. (Balance Sheet) (Income Statement) Supplies Supplies ExpenseTB 599 Bal.___2. Amount of supplies used is $235. (Balance Sheet) (Income Statement) Supplies Supplies ExpenseTB 470Bal.___ ANALYSIS OF ADJUSTING ENTRY FOR INSURANCE: Analyze each situation and indicate the correct dollar amount for the adjusting entry 1. Amount of insurance expired is $970. (Balance Sheet) (Income Statement) Prepaid Insurance Insurance Expense TB 1,450 2. Amount of unexpired insurance is $565. (Balance Sheet) (Income Statement) Prepaid Insurance Insurance Expense TB 1,350 POSTING ADJUSTING ENTRIES: Two adjusting entries are in the following general journal. Post these adjusting entries to the four general ledger accounts. The following account numbers were taken from the chart of accounts: 141, Supplies; 219, Wages Payable 511, Payable; 511, working pan Wages Expense; and S23, Supplies Expense. If you are nor using the he entrs: Supolt accompany this text, enter the following balances before posting s that es: Supplies, $200 Dr ies, $200 Dr; Wages Expense, $1,200 Dr. rent alleles for that characteristic.Which trait did heterozygous individuals show in Mendel's experiments onpea plants?O A. The recessive traitB. The codominant traitC. The dominant traitO D. The acquired trait Write each as an algebraic expression 1) the difference of 10 and 53) u decreased by 175) x increased by 67) the sum of q and 89) twice q11) the quotient of 18 and n help me with these!!!!!!!!!!!!!!!!!!!!! 0The circumference of a circle is 9.22cmFind the length of the diameter.Give your answer rounded to 2 DP.decm Hello I need the answer fast plssssss I will give brianliest!! 20 points. Look at the picture for the question How would Churchill want England to respond to Germanys invasion of Poland? The question is in the screenshot. ( I really need help in this one) Find the average of the numbers 8, 13,15 when monomers are joined together, what do they form? How could you use your technology skill and Microsoft Excel to organize, analyze, and compare data to decide if a specific insurance is a good value for the example you gave Meaning of immortality?? how do the underlined words affect the meaning of the passage What are action-reaction force pairs?